You are on page 1of 16

Natural Resources and Environmental Law with Land Titles and Deeds

Additional Cases assigned by ASG Galandines [CASE DIGESTS]

Republic vs. Court of Appeals Republic, the land recovered would not be for public use, but for
G.R. No. 106763, May 9, 2001 eventual disposition to other private persons. The possessors of the
land would acquire title to the portions they adversely possess through
FACTS: The Municipality of Bacoor, Cavite sold several parcels of acquisitive prescription, without need of title or good faith, pursuant to
land to private respondents predecessors-in-interest. Prior to the sale, the Civil Code.
private respondents predecessor-in-interest, Brigida Francisco, had
been in possession of the subject lot and paid the real estate taxes Republic vs. Court of Appeals
thereon as early as 1907. Private respondents filed an application for G.R. No. L-60078, October 30, 1987
land registration before the CFI of Cavite and a decision was rendered
in favor of the applicant. Pursuant to said judgment, Decree No. N- FACTS: In its application for registration before the trial court,
105464 and the corresponding Original Certificate of Title No. O-468 respondent Circulo Bantayano Foundation, Inc. alleged that it is the
were issued on October 7, 1965. owner in fee simple or thru a possessory information title of a parcel
of land including the buildings and improvements thereon thru
25 years later, the OSG filed with the Court of Appeals a petition to purchase on December 5, 1974 from the heirs (represented by
annul the decision and the OCT; and to order the restoration or Anunciacion Escario) of the late Pedro Escario, Sr. who in turn
reversion of the subject parcels of land to the mass of the public inherited said land from his father Margarito; that the said land is
domain. The OSG alleged that the registration proceedings were null assessed for taxation purposes of P7,850.00 for the year 1978; that the
and void for lack of jurisdiction because the parcels of land subject same is occupied and possessed openly, continuously, notoriously and
thereof were still classified as forest land, having been released peacefully in the concept of owners for more than 40 years by applicant
therefrom only on February 21, 1972. The Court of Appeals ruled that and its predecessors-in-interest. Petitioner opposed the application
by virtue of Act 3312 enacted by the Philippine Legislature, the subject alleging that private respondent did not have title in fee simple or
lots were previously classified as communal in character; that the imperfect title to the land and it was disqualified under the 1973
predecessors-in-interest of private respondents were able to purchase Constitution, being a corporation, to own lands of the public domain.
subject lots as authorized by said Act. Applicant corporation presented in court Tax in the name of the
previous owner Pedro.
ISSUE: Whether the sale of communal lands of the Municipality of
Bacoor to private respondents predecessors-in-interest was authorized The trial court found that the applicant's possession had always been
by law. peaceful, open, public, continuous, notorious and in the concept of
absolute owner thereof and including their predecessors-in-interest's
HELD: YES. We agree with the Court of Appeals that Act No. 3312, possession extended for more than thirty (30) years: that the applicant
known as the Act Authorizing the Sale of the Communal Lands in the herein had been paying regularly the taxes due on the property having
Municipality of Bacoor, Province of Cavite, and for other purposes, declared the land for taxation purposes in its name: that there are no
was the law governing the sale and acquisition by private respondents traversing roads (either national or provincial) nor any river or creek
of the subject parcels of land. ACT 3312 authorized and recognized the crossing the land applied for, and that the produce of the land has been
sale of communal lands in the municipality of Bacoor, Cavite, to solely enjoyed by said applicant.
qualified applicants provided the (1) buyers should be tenants and
lessees as of December 2, 1926 of the different parcels composing said ISSUES: Whether or not private respondent is qualified under the
lands as recorded in the communal lands register in the Office of the 1973 or the 1987 Constitutions to acquire and subsequently register in
Mayor; and (2) they must pay the corresponding rental in arrears and its name the disputed lot.
other obligations due to the municipality.
HELD: It is true that under both the 1973 and the 1987 Constitution,
Secondly, it is error to say that private respondents, as qualified 3 3 a private corporation (even if a domestic one) cannot acquire (and
applicants, could no longer avail of the benefits of Act 3312, due to the therefore cannot register) lands of the public domain, but in the present
subsequent passage of C.A. 141. True, the sale was perfected and case the land involved, at the time it was acquired by the corporation
consummated only on April 22, 1964 and the application for in 1974, was no longer part of the public domain; long years of
registration was filed only on October 27, 1964. But we agree with the exclusive continuous, and adverse possession of the same by its
Court of Appeals that the right of private respondents had already predecessors-in-interest had given ownership thereof ipso jure to said
acquired the sanctity of vested or acquired rights which cannot be predecessors, enabling the latter to convey title to said corporation.
defeated or adversely affected by the passage of C.A. 141. True, the Corporation's acquisition was in 1974, or after the 1973 was
already in effect. But then as of that time, the land was no longer public
Thirdly, we are not persuaded by petitioners argument that Act 3312 land. It was private land. Thus, the prohibitions referred to in the 1973
was repealed by C.A. 141. The Public Land Act is a general law and 1987 Constitutions can no longer apply.
governing the administration and disposition of the lands of the public
domain; while Act 3312 is a special law on the sale of lands designated Republic vs. Court of Appeals
as communal situated in the Municipality of Bacoor in Cavite. The G.R. No. L-40402, March 16, 1987
presumption against implied repeal is stronger when of two laws, one
is special and the other general; and this rule applies even though the FACTS: Lot No. 622 of the Mariveles Cadastre was declared public
terms of the general act are broad enough to include the matter covered land in a decision rendered in Cadastral Case No. 19. Lot 622 was
by the special statute. segregated from the forest zone and released and certified by the
Bureau of Forestry as an agricultural land for disposition under the
Finally, the Republic as petitioner, does not stand to be deprived of its Public Land Act. Respondents filed in the Court of First Instance of
patrimony, as the said parcels of land had already been declared Bataan a petition to reopen Cadastral Case No. 19 under Republic Act
alienable and disposable and if there is any reversion in favor of the 931, as amended by Republic Act 2061, concerning a portion of Lot

ADDITIONAL CASES IN NATURAL RESOURCES & ENVIRONMENTAL LAW WITH LAND TITLES AND DEEDS [CASE DIGESTS] JAMES BRYAN DEANG
No. 622 and Lot No. 324 to perfect their rights and register their titles appeal should be reckoned from the time the Solicitor-
to said lots, having allegedly acquired ownership and possession of General's Office is apprised of the 1970 order of denial and
said parcels of land by purchase from the original owners thereof, not from the time the special counsel or the fiscal was served
whose possession of the same including that of the herein Respondents, with that order. Thus, representatives of the Solicitor
has always been continuous, open, active, exclusive, public, adverse, General in the case at bar, had no power to decide whether
and in the concept of owners thereof for more than 30 years. or not an appeal should be made. They should have referred
the matter to the Solicitor-General and without copies of
the lower court issued an Order setting the petition for hearing and court orders, notices and decisions, having been provided by
directing that the Republic of the Philippines be notified thereof. The either the trial court or the Provincial Fiscal of Bataan to the
Director of Forestry filed an opposition to the petition praying for the Solicitor-General, the assailed decision has no binding effect
denial of the petition once the area involved is found to be within the on the government.
timberland and therefore inalienable under the Constitution. Upon
verification, however, the Director of Forestry found the area to be the Republic vs. Court of Appeals
portion of the timberland already released by the government from the G.R. No. 113549, July 5, 1996
mass of public forests and promptly withdrew his Opposition. The
Acting Provincial Fiscal of Bataan, for and in behalf of the Director of FACTS: A parcel of land located in the barrio of Magragondong,
Lands, filed his opposition to the petition alleging that the land is still, Municipality of Ligao, Province of Albay, was surveyed for the
in truth and in fact, public land and as such cannot be the subject of a spouses Ribaya. The survey plan was denominated as Plan II-13961
land registration proceeding under Act 496. The lower court found that and allegedly approved by the Acting Director of Lands. However, as
the petitioners have complied with all the terms and conditions which noted by the Court of Appeals, these exhibits do not at all show the
would entitle them to a grant. surveyor's signature. Moreover, as per Land Classification Map No.
871 of the Bureau of Forestry, the above parcel of land was considered
ISSUE: part of the public forest and released for disposition only on 31
1. Whether or not the lots claimed by respondents could legally December 1930. In 1925, the spouses Ribaya applied for registration
be the subject of a judicial confirmation of title under the and confirmation of title of the lot. The CFI granted the said
aforequoted provisions of the Public Land Act, as amended. application.

2. Whether or not THE RESPONDENT COURT OF In November 1925, a resurvey of the parcel of land was conducted at
APPEALS COMMITTED A REVERSIBLE ERROR IN the instance of the spouses Ribaya. This gave rise to an amended plan
NOT HOLDING THAT THE ENTIRE PROCEEDING embracing four different parcels of land. The appeared to have been
FOR REOPENING OF THE CADASTRAL CASE OVER approved by the Director of Lands. The application was not amended
THE LOTS IN QUESTION WAS VITIATED BY LACK to reflect the resurvey and the amended plan was not published. An
OF NOTICE TO THE SOLICITOR-GENERAL. OCT covering the four lots was issued in the names of the spouses
Ribaya. In 1958, the OCT was administratively reconstituted from the
HELD: owner's duplicate copy thereof and the reconstituted title was
1. NO. Section 48(b) of C.A. No. 141, as amended, applies denominated as OCT No. RO-10848 (3947).
exclusively to public agricultural land. Forest lands or areas
covered with forests are excluded. They are incapable of In 1964, the heirs of Luis Ribaya received compensation from the
registration and their inclusion in a title, whether such title Foreign Claims Settlement Commission of the United States for
be one issued during the Spanish sovereignty or under the damages sustained by the land during the war. In 1968, pursuant to a
present Torrens system of registration, nullifies the title. deed of partition executed by the private respondents herein, the land
Thus, possession of forest lands, however long, cannot ripen covered by OCT No. RO-10848 (3947) was subdivided. Then, OCT
into private ownership. A parcel of forest land is within the No. RO-10848 (3947) was cancelled and separate Transfer Certificates
exclusive jurisdiction of the Bureau of Forestry and beyond of Title (TCT) were issued to the private respondents.
the power and jurisdiction of the cadastral court to register
under the Torrens System. Thus, even if the reopening of the 62 farmers occupying the land and claiming ownership thereof,
cadastral proceedings was at all possible, private requested the Director of Lands to institute an action to annul OCT No.
respondents have not qualified for a grant under Sec. 48(b) RO-10848 (3947). Finding merit in the request, herein petitioner filed
of Commonwealth Act 141, the facts being that private a verified complaint for the declaration of nullity of OCT No. 3947,
respondents could only be credited with 1 year, 9 months and OCT No. RO-10848 (3947), and all subsequent titles emanating from
20 days possession and occupation of the lots involved, the original title. The petitioner claimed therein that OCT No. 3947
counted from July 6, 1965, the date when the land area had was obtained through fraud and that the land registration court did not
been segregated from the forest zone and released by the acquire jurisdiction over the land for lack of republication of the
Bureau of Forestry as an agricultural land for disposition amended plan, neither did the spouses-applicants comply with Section
under the Public Land Act. Consequently, under the above 45(b) of Act No. 2874. The petitioner further alleged that at the time
mentioned jurisprudence, neither private respondents nor the petition for registration was filed, the land covered therein was
their predecessors-in-interest could have possessed the lots forest land, and therefore, inalienable.
for the requisite period of thirty (30) years as disposable
agricultural land. ISSUES:
1. Whether the Republic of the Philippines is barred by prescription
2. YES. In a more recent case, Republic v. Court of Appeals, to bring the action for annulment of OCT No. 3947 and all its
135 SCRA 161 [1985], it was established that the Solicitor- derivative certificates of title.
General is the only legal counsel of the government in land
registration cases and as such, he alone may withdraw the 2. Whether the land registration court acquired jurisdiction over the
Government's appeal with binding effect on the latter. He is four parcels of land subject of the amended survey plan and
entitled to be furnished copies of all court orders, notices and covered by the decree issued on 31 July 1926 by the General Land
decisions and as held the reglementary thirty-day period for
ADDITIONAL CASES IN NATURAL RESOURCES & ENVIRONMENTAL LAW WITH LAND TITLES AND DEEDS [CASE DIGESTS] JAMES BRYAN DEANG
Registration Office pursuant to the decision of the said court of 18 HELD: NO. The contention has no merit. The provision of the
September 1925. Administrative Code relied upon by the Solicitor General is not new.
It is simply a codification of 1(e) of P.D. No. 478 (Defining the
HELD: Powers and Functions of the Office of the Solicitor General). It is only
1. NO. First, the one-year period provided for in Section 38 of Act now that the Solicitor General is claiming the right to be notified of
No. 496 merely refers to a petition for review and is reckoned from proceedings for the issuance of the owner's duplicate certificate of title.
the entry of the decree. In the second place, there are other Indeed, the only basis for such claim is that the Office of the Solicitor
remedies available to an aggrieved party after the said one-year General represents the government in land registration and related
period, e.g., reconveyance, covered by Section 65 of Act No. 496 proceedings. Even so, however, the request for representation should
which provides that "in all cases of registration procured by fraud, have come from the Registrar of Deeds of Makati who was the proper
the owner may pursue all his legal and equitable remedies against party to the case. Here, there is no dispute that the Registrar of Deeds
the parties to such fraud, without prejudice, however, to the rights of Makati was notified of private respondent's petition, but she
of any innocent holder for value of a certificate of title." Likewise, manifested that her office had no objection thereto. The Solicitor
an action for damages is sanctioned in cases where the property General does not question the propriety of the action and manifestation
has been transferred to an innocent purchaser for value, which may of the Registrar of Deeds, nor does he give any reason why private
be led within four years from discovery of the fraud. Recourse may respondent's petition for the issuance of a new owner's duplicate
also be had against the Assurance Fund. Finally, prescription certificate of title should be denied. Instead, he claims that the fact that
never lies against the State for the reversion of property which is he was given a copy of the decision is an admission that he is entitled
part of the public forest or of a forest reservation which was to be notified of all incidents relating to the proceedings. This is not
registered in favor of any party. Then too, public land registered correct. Considering that the law does not impose such notice
under the Land Registration Act may be recovered by the State at requirement in proceedings for the issuance of a new owner's duplicate
any time. certificate of title, the lack of notice to the Solicitor General, as counsel
for the Registrar of Deeds, was at most only a formal and not a
2. Anent the second issue, we hold that the land registration court jurisdictional defect.
never acquired jurisdiction over the land covered by either the
original plan or the amended plan for lack of sufficient publication This case should be distinguished from our rulings in cadastral
of the first and total want of publication of the second. There was registration cases and original land registration proceedings which
only one publication thereof. Section 31 of Act No. 496, the require that the Solicitor General be notified of decisions and hold as
governing law then, required two publications. Hence, the decision decisive, for the purpose of determining the timeliness of the appeal
of 18 September 1925 of the land registration court was void for filed by the government, the date of his receipt of the decisions therein
want of the required publications. The requirement of dual and not that of the Director of Lands or of his other representatives.
publication is one of the essential bases of the jurisdiction of the The issue and the applicable laws in those cases are different. The
registration court; it is a jurisdictional requisite. Land registration important role of the Office of the Solicitor General as the
is a proceeding in rem and jurisdiction in rem cannot be acquired government's law office cannot be overemphasized. Its powers and
unless there be constructive seizure of the Land through functions, however, should not be rigidly applied in such a manner that
publication and service of notice. innocuous omissions, as in the case at bar, should be visited with so
grave a consequence as the nullification of proceedings. After all, no
Republic vs. Court of Appeals prejudice to the government has been shown.
G.R. No. 128531, October 26, 1999
Del Rosario-Igbiten vs. Republic
FACTS: Vicente Yupangco is the owner of a unit in a condominium G.R. No. 158449, October 22, 2004
building as evidenced by Certificate of Title No. 7648. Because his
aforesaid certificate could not be located, he filed a petition for the FACTS: Petitioners filed with the trial court an application for
issuance of a new duplicate certificate of title in lieu of his lost copy, registration of land under Presidential Decree (PD) No. 1529. The
pursuant to 109 of P.D. No. 1529 (Property Registration Decree). The application covered Lot 5442, Cad 452-D, Silang Cadastre, Ap-04-
trial court ordered the Registrar of Deeds of Makati to comment on the 007007. Petitioners alleged that they acquired the Subject Property by
petition and thereafter set the case for initial hearing. The Registrar of purchase, and that they, by themselves and through their predecessors-
Deeds of Makati filed a manifestation that she had no objection to the in-interest, had been in actual, continuous, uninterrupted, open, public,
petition. After hearing private respondent's evidence, the trial court and adverse possession of the Subject Property in the concept of owner
rendered its decision granting the petition, declaring as invalid the for more that 30 years. No opposition was filed against the application
missing copy of the certificate of title, and ordering the Registrar of and so petitioners proceeded with the presentation of their evidence.
Deeds of Makati to issue a new owner's duplicate certificate of title in The State was represented in the proceedings by Assistant Provincial
the name of private respondent. A copy of this decision was furnished Prosecutor Jose M. Velasco, Jr. Based on the testimonial and
the Solicitor General. The Solicitor General moved for reconsideration documentary evidence presented, the trial court traced the history of
of the trial court's decision on the ground that no copy of private possession of the Subject Property back to 1958, when the Subject
respondent's petition or notice thereof had been given to him. His Property was first declared for tax purposes by Justina Hintog. Teodoro
motion was, however, denied. The Office of the Solicitor General then Calanog came into possession of the Subject Property in 1968. In the
elevated the case to the Court of Appeals, which, in a affirmed the same year, the Subject Property was transferred to spouses Alfredo
order of the trial court. Tonido and Agatona Calanog. Agatona Calanog allegedly inherited the
Subject Property from Teodoro Calanog, her father; on the other hand,
ISSUE: Whether or not it was mandatory for the trial court to notify Alfredo Tonido supposedly purchased the same property also from
him [OSG] of private respondent's petition and that its failure to do so Teodoro Calanog, his father-in-law. Alfredo Tonido planted the
rendered the proceedings before it null and void. Subject Property with palay, sayote, coffee, guyabano and other fruit
bearing trees.

ADDITIONAL CASES IN NATURAL RESOURCES & ENVIRONMENTAL LAW WITH LAND TITLES AND DEEDS [CASE DIGESTS] JAMES BRYAN DEANG
After the demise of Agatona Calanog, the rest of the Tonido family, Counsel for oppositors failed to appear on two different hearings
consisting of Alfredo and his children, Samuel, Elizabeth, Benjamin, despite due notice. Hence, the court issued an order submitting the case
Imelda and Esther, shared possession of the Subject Property. On 21 for decision based on the evidence of the petitioners. Accordingly, the
November 1995, the Tonido family sold the Subject Property to court rendered a decision confirming the imperfect title of petitioners.
petitioners, as evidenced by a Deed of Absolute Sale. The trial court
rendered a decision approving petitioners' application. In 1991, the Solicitor General filed with the Court of Appeals a Petition
for Annulment of Judgment pursuant to Section 9(2) of BP Blg. 129
The Office of the Solicitor General, appealed the decision of the trial on the ground that there had been no clear showing that the Lot had
court to the Court of Appeals. In its appeal, the Republic alleged that been previously classified as alienable and disposable. Also,
the trial court erred in approving the application for registration despite Bockasanjo ISF Awardees Association, Inc., an association of holders
petitioners' failure to prove open, continuous, exclusive and notorious of certificates of stewardship issued by the DENR under its Integrated
possession and occupation of the Subject Property since 12 June 1945, Social Forestry Program filed with the Court of Appeals a Motion for
or earlier, as required by Section 48(b) of Commonwealth Act No. 141, Leave to Intervene and to Admit Petition-In-Intervention. They
as amended by PD No. 1073. Moreover, petitioners also failed to likewise opposed the registration and asserted that the Lot, which is
produce muniments of title to tack their possession to those of their situated inside the Marikina Watershed Reservation, is inalienable.
predecessors-in-interest in compliance with the prescriptive period They claimed that they are the actual occupants of the Lot pursuant to
required by law. The Court of Appeals rendered a decision finding the the certificates of stewardship issued by the DENR under the ISF for
appeal meritorious, setting aside the decision of the trial court, and tree planting purposes. The Court of Appeals granted the petition and
dismissing the application for registration of petitioners. declared null and void the decision of the land registration court.

ISSUE: Whether or not petitioners have failed to comply with the ISSUES:
period of possession and occupation of the Subject Property, as 1. Whether the petitioners have registrable title over the land.
required by both the Property Registration Decree and the Public Land
Act. HELD:
1. NONE. There is no dispute that EO 33 established the Marikina
HELD: YES. Watershed Reservation situated in the Municipality of Antipolo,
Indeed, the earliest period that the applicants could claim ownership Rizal. Petitioners even concede that the Lot, described as Lot Psu
over the property is in 1958, which is the earliest date Justina Hintog, 162620, is inside the technical, literal description of the MWR.
the previous owner/occupant, declared the property for taxation However, the main thrust of petitioners' claim over the Lot is that
purposes. This is far later than June 12, 1945, the date prescribed by "all Presidential proclamations like the proclamation setting aside
law that the applicants' possession under claim of ownership should the Marikina Watershed Reservation are subject to private rights."
have begun at the latest. They point out that EO 33 contains a saving clause that the
reservations are "subject to existing private rights, if any there be."
Petitioners maintain, however, that RA No. 6940, enacted on 28 March Petitioners contend that their claim of ownership goes all the way
1990, has repealed by implication Section 48(b) of the Public Land back to 1902, when their known predecessor-in-interest, Sesinando
Act, as amended by PD No. 1073, and has effectively reduced the Leyva, laid claim and ownership over the Lot. They claim that the
required period of possession and occupation of the land to thirty years presumption of law then prevailing under the Philippine Bill of
prior to the filing of the application for confirmation of an imperfect or 1902 and Public Land Act No. 926 was that the land possessed and
incomplete title. Petitioners' arguments are without merit. claimed by individuals as their own are agricultural lands and
therefore alienable and disposable. They conclude that private
IN ALL: (1) Section 44 of the Public Land Act, as amended by RA No. rights were vested on Sesinando Leyva before the issuance of EO
6940, which provides for a prescriptive period of thirty (30) years 33, thus excluding the Lot from the Marikina Watershed
possession, applies only to applications for free patents; (2) The case Reservation. Petitioners' arguments find no basis in law.
at bar is a judicial application for confirmation of an imperfect or
incomplete title over the Subject Property covered by Section 48(b) of Sta. Rosa Realty gives us a glimpse of the dangers posed by the misuse
the Public Land Act; and (3) Section 48(b) of the Public Land Act of natural resources such as watershed reservations which are akin to
requires for judicial confirmation of an imperfect or incomplete title forest zones. Population growth and industrialization have taken a
the continuous possession of the land since 12 June 1945, or earlier, heavy toll on the environment. Environmental degradation from
which petitioners herein failed to comply with. unchecked human activities could wreak havoc on the lives of present
and future generations. Hence, by constitutional fiat, natural resources
Collado vs. Court of Appeals remain to this day inalienable properties of the State. Viewed under
G.R. No. 107764, October 4, 2002 this legal and factual backdrop, did petitioners acquire, as they
vigorously argue, private rights over the parcel of land prior to the
FACTS: Edna T. Collado filed with the land registration court an issuance of EO 33 segregating the same as a watershed reservation?
application for registration of a parcel of land covered by Survey Plan The answer is in the negative.
Psu162620. Attached to the application was the technical description
of the Lot signed by the Officer-in-Charge of the Survey Division, Assuming that the Lot was alienable and disposable land prior to the
Bureau of Lands. In 1986, Edna T. Collado filed an Amended issuance of EO 33 in 1904, EO 33 reserved the Lot as a watershed.
Application to include additional co-applicants. Subsequently, more Since then, the Lot became nondisposable and inalienable public land.
applicants joined. The Republic of the Philippines filed. In due course, At the time petitioners filed their application on April 25, 1985, the Lot
the land registration court issued an order of general default against the has been reserved as a watershed under EO 33 for 81 years prior to the
whole world with the exception of the oppositors. Petitioners alleged filing of petitioners' application. The period of occupancy after the
that they have occupied the Lot since time immemorial. Their issuance of EO 33 in 1904 could no longer be counted because as a
possession has been open, public, notorious and in the concept of watershed reservation, the Lot was no longer susceptible of occupancy,
owners. Petitioners declared the Lot for taxation purposes and paid all disposition, conveyance or alienation.
the corresponding real estate taxes. According to them, there are now
twenty-five co-owners in pro-in-diviso shares of five hectares each.
ADDITIONAL CASES IN NATURAL RESOURCES & ENVIRONMENTAL LAW WITH LAND TITLES AND DEEDS [CASE DIGESTS] JAMES BRYAN DEANG
Republic vs. Doldol ISSUES:
G.R. No. 132963, September 10, 1998 1. Whether respondents were able to prove that the lots subject
of their application were then disposable and alienable land
FACTS: Nicanor Doldol occupied a portion of land in Barrio of the public domain; and
Pantacan, Municipality of Opol, Misamis Oriental in 1959. He filed an 2. Whether they were able to show that they have been in open,
application for saltwork purposes for the said area with the Bureau of exclusive, continuous and notorious possession of the lots in
Forest Development on October 23, 1963, but it was rejected on April the concept of owners.
1, 1968. However, in 1965, the Provincial Board of Misamis Oriental
passed a resolution reserving lot 4932, Cad-237 Opol Cadastre which HELD:
included the area occupied by Doldol as a school site. Accordingly, in 1. YES.
1970, the Opol High School, now called as Opol National Secondary Accordingly, applicants for registration of land must prove:
Technical School, transferred to the said area. And on November 2, (a) that it forms part of the alienable lands of the public domain; and
1987, President Corazon Aquino issued Proclamation No. 180 (b) that they have been in open, exclusive, continuous and notorious
reserving the said area for the said school. The school, then, made possession and occupation of the same under a bona fide claim of
several demands to Doldol to vacate the portion occupied by him but ownership either since time immemorial or since June 12, 1945.
he refused to do so. As a consequence, an accion possessoria was filed
by the school against him, and the Regional Trial Court of Cagayan de Relative to the first requisite, it is undisputed that the subject lots have
Oro ruled in the school's favor and ordered him to vacate the land. On been declared as alienable and disposable by a positive government
appeal, the Court of Appeals reversed the decision of the court a quo . act. The evidence on record shows that a Certification was issued by
Hence, this petition. the CENRO, DENR to the effect that the lots are within Camarines
Norte Project No. 4-E, alienable and disposable area.
ISSUE: Whether or not the Court of Appeals erred on a question of
law when it held, contrary to the evidence on record, that respondent 2. NO
had been in open, continuous, notorious and exclusive possession of Here, when respondents filed their application in 1994, the lots were
the land in dispute for thirty-two years. already declared alienable and disposable by the DENR 49 years ago,
or in 1945. Unfortunately, evidence on record shows that their
HELD: The petition is meritorious. The original Section 48 (b) of C.A. predecessors-in-interest have been in open, exclusive and continuous
No. 141 provided for possession and occupation of lands of the public possession of the disputed property only since 1955. Although
domain since July 16, 1894. This was superseded by R.A. No. 1942, respondents possession and that of their predecessors-in-interest was
which provided for a simple thirty year prescriptive period of more than 39 years when they filed their application for registration in
occupation by an applicant for judicial confirmation of imperfect title. 1994, that period of possession will not suffice for purposes of
The same, however, has already been amended by Presidential Decree registration of title. What is required is open, exclusive, continuous and
No. 1073, approved on January 25, 1977. The parties, however, notorious possession by respondents and their predecessors-in-interest,
stipulated during the pre-trial hearing that Doldol had been occupying under a bona fide claim of ownership, since June 12, 1945 or earlier.
the portion reserved for the school site only since 1959. The law, as
presently phrased, requires that possession of lands of the public Tan vs. Republic
domain must be from June 12, 1945 or earlier, for the same to be G.R. No. 177797, December 4, 2008
acquired through judicial confirmation of imperfect title.
Consequently, Doldol could not have acquired an imperfect title to the FACTS: The spouses Tan were natural-born Filipino citizens, who
disputed lot since his occupation of the same started only in 1959, became Australian citizens. They seek to have the subject property
much later than June 12, 1945. Not having complied with the registered in their names. The subject property was declared alienable
conditions set by law, Doldol cannot be said to have acquired a right and disposable on 31 December 1925, as established by a Certication
to the land in question as to segregate the same from the public domain. issued DENR, CENRO, Cagayan de Oro City. Prior to the spouses
Doldol cannot, therefore, assert a right superior to the school. In sum, Tan, the subject property was in the possession of Lucio and Juanito
Opol National School has the better right of possession over the land Neri and their respective spouses. Lucio and Juanito Neri had declared
in dispute. the subject property for taxation purposes in their names since 1952.
The spouses Tan acquired the subject property from Lucio and Juanito
Republic vs. Bibonia Neri and their spouses by virtue of a duly notarized Deed of Sale of
G.R. No. 157466, June 21, 2007 Unregistered Real Estate Property. The spouses Tan took immediate
possession of the subject property on which they planted rubber,
FACTS: Cheryl B. Bibonia, and Fernandez, substituted by Manahan, gemelina, and other fruit-bearing trees.
filed an application for registration of two parcels of land. Respondent
Bibonias application covers Lot 1, while that of respondent Manahan They declared the subject property for taxation purposes in their names
covers Lot 2. Both lots are located in Vinzons, Camarines Norte. since 1971 and paid realty taxes thereon. However, a certain Casio
Bibonia alleged in her application that she bought Lot 1 from Marita claimed a portion of the subject property, prompting the spouses Tan
King as shown by a Deed of Absolute Sale. Marita, in turn, received to file a Complaint for Quieting of Title against him. The RTC
the property from her father, Mariano Morales, by virtue of a Deed of rendered a favoring the spouses Tan. Casio appealed to the Court of
Donation Inter Vivos. Morales, on the other hand, purchased the same Appeals which dismissed the case. Casio elevated his case to this
lot from Barco who, in turn, bought it from Restituto Paraon in 1955. Court via a Petition for Review on Certiorari which denied Casio's
Petition for being insufficient in form and substance. The said
Manahan, alleged that he bought Lot 2 from Cribe. It was donated to Resolution became final and executory. Refusing to give up, Casio
the latter by her father Morales who purchased the same lot from Barco led an Application for Free Patent on the subject property before the
and the heirs of Salome in 1968. The trial court granted respondents Bureau of Lands but was cancelled upon the request of herein
application for registration. On appeal by the Republic, the Court of petitioner Pedro Tan, the declared owner of the subject property.
Appeals affirmed in toto the trial courts judgment.

ADDITIONAL CASES IN NATURAL RESOURCES & ENVIRONMENTAL LAW WITH LAND TITLES AND DEEDS [CASE DIGESTS] JAMES BRYAN DEANG
The spouses Tan filed their Application for Registration of Title to the are now asking the kind indulgence of this Court to take into account
subject property. The application of the spouses Tan invoked the Tax Declaration No. 4627 issued in 1948, which they had attached to
provisions of Act No. 496 and/or Section 48 of Commonwealth Act their Motion for Reconsideration before the Court of Appeals but
No. 141, as amended. In compliance with the request of the LRA, the which the appellate court refused to consider. Just as they had argued
spouses Tan filed an Amended Application for Registration of Title to before the Court of Appeals, the spouses Tan point out that Tax
the subject property. The OSG entered its appearance but failed to Declaration No. 4627 was not newly issued but cancelled Tax
submit a written opposition to the application of the spouses Tan. The Declaration No. 2948; and should the Court take judicial notice of the
RTC issued an order of general default, except as against the Republic. fact that tax assessments are revised every four years, then Tax
Declaration No. 2948 covering the subject property was issued as early
After the establishment of the jurisdictional facts, the RTC heard the as 1944.
testimony of Acero, nephew and lone witness of the spouses Tan.
After Acero's testimony, the spouses Tan already made a formal offer In this case, it bears stressing that Tax Declaration No. 4627 was only
of evidence, which was admitted by the court a quo which rendered a submitted by the Spouses Tan together with their Motion for
Decision granting the application of the spouses Tan. On Appeal, the Reconsideration of the 28 February 2006 Decision of the Court of
Court of Appeals rendered a Decision granting the appeal of the Appeals. The reason given by the Spouses Tan why they belatedly
Republic, and reversing and setting aside the Decision of the RTC on procured such evidence was because at the time of trial the only
the ground that the spouses Tan failed to comply with Section 48 (b) evidence available at hand was the 1952 tax declaration. More so, they
of Commonwealth Act No. 141. also believed in good faith that they had met the 30-year period
required by law. They failed to realize that under Section 48 (b) of
ISSUE: Whether or not the Spouses Tan have been in open, Commonwealth Act No. 141, as amended, a mere showing of
continuous, exclusive, and notorious possession and occupation of the possession for thirty years or more is not sufcient because what the law
subject property, under a bona fide claim of acquisition or ownership, requires is possession and occupation on or before 12 June 1945. This
since 12 June 1945, or earlier, immediately preceding the filing of the Court, however, nds the reason given by the spouses Tan
application for confirmation of title. unsatisfactory. The spouses Tan filed their application for registration
of title to the subject property under the provisions of Section 48 (b) of
HELD: Commonwealth Act No. 141, as amended. It is incumbent upon them
The Court notes that Presidential Decree No. 1073, amending the as applicants to carefully know the requirements of the said law. Thus,
Public Land Act, claried Section 48, paragraph "b" thereof, by following the rule enunciated in Section 34, Rule 132 of the Rules of
specifically declaring that it applied only to alienable and disposable Court, this Court cannot take into consideration Tax Declaration No.
lands of the public domain. Thus, based on the said provision of 4627 as it was only submitted by the Spouses Tan when they led their
Commonwealth Act No. 141, as amended, the two requisites which the Motion for Reconsideration of the 28 February 2006 Decision of the
applicants must comply with for the grant of their Application for appellate court.
Registration of Title are: (1) the land applied for is alienable and
disposable; and (2) the applicants and their predecessors-in-interest Republic vs. Manna Properties
have occupied and possessed the land openly, continuously, G.R. No. 146527, January 31, 2005
exclusively, and adversely since 12 June 1945.
FACTS: Applicant-appellee filed an Application for the registration
To prove that the land subject of an application for registration is of title of two (2) parcels of land. Initial hearing was set on February
alienable, an applicant must conclusively establish the existence of a 16, 1995 by the court a quo. Copies of the application, postal money
positive act of the government such as a presidential proclamation or orders for publication purposes and record were forwarded to the Land
an executive order or administrative action, investigation reports of the Registration Authority by the Court a quo on October 7, 1994.
Bureau of Lands investigator or a legislative act or statute. Until then, However, per Report of the Land Registration Authority, the full
the rules on confirmation of imperfect title do not apply. names and complete postal addresses of all adjoining lot owners were
not stated for notification purposes. As a result thereto, the applicant
On the first requisite was directed to submit the names and complete postal addresses of the
In the case at bar, the spouses Tan presented a Certication from the adjoining owners of Lots 9514 and 9516. The applicant filed its
DENR CENRO, Cagayan de Oro City, dated 14 August 2000, to prove compliance, which was forwarded to the LRA together with the notice
the alienability and disposability of the subject property. The said of the Initial Hearing, which was reset to April 13, 1995. The LRA
Certification stated that the subject property became alienable and requested for the resetting of the initial hearing since April 13, 1995
disposable on 31 December 1925. A certification from the DENR that fell on Holy Thursday to a date consistent with LRC Circular No. 353
a lot is alienable and disposable is sufficient to establish the true nature or ninety (90) days from date of the Order to allow reasonable time for
and character of the property and enjoys a presumption of regularity in possible mail delays and to enable them to cause the timely publication
the absence of contradictory evidence. Considering that no evidence of the notice in the Official Gazette. The initial hearing was,
was presented to disprove the contents of the aforesaid DENR-CENRO accordingly. The court a quo received a letter dated from the LRA with
Certification, this Court is duty bound to uphold the same. the information that the notice can no longer be published in the
Nonetheless, even when the spouses Tan were able to sufficiently Official Gazette for lack of material time since the National Printing
prove that the subject property is part of the alienable and disposable Office required submission of the printing materials 75 days before the
lands of the public domain as early as 31 December 1925, they still date of the hearing. The initial hearing was reset again to July 18, 1995.
failed to satisfactorily establish compliance with the second requisite The Opposition to the application stated, among others, that the
for judicial confirmation of imperfect or incomplete title, i.e., open, applicant is a private corporation disqualified under the new Philippine
continuous, exclusive and notorious possession and occupation of the Constitution to hold alienable lands of public domain. Per Certificate
subject property since 12 June 1945 or earlier. of Publication issued by the LRA and the National Printing Office, the
Notice of Initial Hearing was published. The same notice was
On the second requisite published in the The Ilocos Herald. Applicant-appellee presented its
The earliest evidence of possession and occupation of the subject president Jose [Tanyao], who testified on the acquisition of the subject
property can be traced back to a tax declaration issued in the name of property as well as Manuel Sobrepea, co-owner of the subject
their predecessors-in-interest only in 1952. However, the spouses Tan property, who testified on the possession of the applicant-appellee's
ADDITIONAL CASES IN NATURAL RESOURCES & ENVIRONMENTAL LAW WITH LAND TITLES AND DEEDS [CASE DIGESTS] JAMES BRYAN DEANG
predecessors-in-interest. Thereafter, the court a quo rendered a
Decision granting the application. The Office of the Solicitor General We rule, however, that the land in question has not become private land
promptly appealed to the Court of Appeals which dismissed and remains part of the public domain. Although Manna Properties
petitioner's appeal. claimed during trial that they were presenting the tax declaration
proving possession since 12 June 1945, a scrutiny of the tax declaration
ISSUE: reveals that it is not the tax declaration Manna Properties claimed it to
1. WHETHER MANNA PROPERTIES FAILED TO be. Exhibit Q - 16 was in fact substitute tax declaration allegedly
COMPLY WITH THE JURISDICTIONAL issued on 28 November 1950. The annotation at the back of this tax
REQUIREMENTS FOR ORIGINAL REGISTRATION. declaration indicates that it was issued to replace the 1945 tax
declaration covering the land in question. A substitute is not enough.
2. WHETHER MANNA PROPERTIES HAS
SUFFICIENTLY PROVEN POSSESSION OF THE Recto vs. Republic
PROPERTY FOR THE REQUISITE PERIOD. G.R. No. 160421, October 4, 2004

HELD: FACTS: Spouses Recto, filed an application for registration of title


1. NO. over Lot 806 under PD No. 1529. They also prayed in the alternative
Petitioner contends that PD 1529 sets a 90-day maximum period that their petition for registration be granted pursuant to CA No. 141.
between the court order setting the initial hearing date and the hearing Petitioners alleged that on June 4, 1996, they purchased Lot 806 from
itself. Petitioner points out that in this case, the trial court issued the sisters Rosita and Maria Medrana who inherited the lot from their
order setting the date of the initial hearing on 15 March 1995, but the deceased parents. Maria and Rosita declared that since 1945, her father
trial court set the hearing date itself on 18 July 1995. Considering that was already the owner of Lot 806. The sisters added that during the
there are 125 days in between the two dates, petitioner argues that the lifetime of Vicente, he planted rice and corn on the lot with the help of
trial court exceeded the 90-day period set by PD 1529. Thus, petitioner their tenant. After his demise, they continued to plant the same crops
concludes "the applicant [Manna Properties] failed to comply with the through hired farmers. There being no opposition to the petition from
jurisdictional requirements for original registration." The petitioner is any private individual, an Order of General Default was issued by the
mistaken. The duty and the power to set the hearing date lies with the trial court which later rendered a decision granting the petition for
land registration court. After an applicant has filed his application, the registration.
law requires the issuance of a court order setting the initial hearing
date. Petitioner is correct that in land registration cases, the applicant The OSG appealed to the Court of Appeals contending that petitioners
must strictly comply with the jurisdictional requirements. failed to (1) offer in evidence the original tracing cloth plan of the land;
(2) prove possession of the lot for the period required by law; and (3)
In this case, the applicant complied with the jurisdictional overthrow the presumption that subject property forms part of the
requirements. The facts reveal that Manna Properties was not at fault public domain. The Court of Appeals reversed the decision on the sole
why the hearing date was set beyond the 90-day maximum period. The ground of failure to offer in evidence the original tracing cloth plan of
records show that the Docket Division of the LRA repeatedly the land.
requested the trial court to reset the initial hearing date because of
printing problems with the National Printing Office, which could affect ISSUE: Whether or not the requirements presented by the petitioners
the timely publication of the notice of hearing in the Official Gazette. for the registration of the title were insufficient and thus the case must
Indeed, nothing in the records indicates that Manna Properties failed be remanded to the court a quo.
to perform the acts required of it by law. A party cannot intervene in
matters within the exclusive power of the trial court. No fault is HELD: NO.
attributable to such party if the trial court errs on matters within its sole Rosita and Maria, the predecessors-in-interest of petitioners,
power. It is unfair to punish an applicant for an act or omission over categorically testified that they, and prior to them their father, had been
which the applicant has neither responsibility nor control, especially if cultivating and possessing Lot 806 in the concept of owners. The trial
the applicant has complied with all the requirements of the law. court found their testimonies to be worthy of belief and credence. The
fact that the earliest Tax Declaration of the subject lot was for the year
2. NO 1948 will not militate against petitioners. It was held that the belated
The issue of whether Manna Properties has presented sufficient proof declaration of the lot for tax purposes does not necessarily mean that
of the required possession, under a bona fide claim of ownership, raises possession by the previous owners thereof did not commence in 1945
a question of fact. It invites an evaluation of the evidentiary record. or earlier. As long as the testimony supporting possession for the
Petitioner invites us to re-evaluate the evidence and substitute our required period is credible, the court will grant the petition for
judgment for that of the trial and appellate courts. Generally, Rule 45 registration.
does not allow this. Matters of proof and evidence are beyond the
power of this Court to review under a Rule 45 petition, except in the So also, there is no doubt that Lot 806 is an alienable land of the public
presence of some meritorious circumstances. We find one such domain having been released and certified as such on December 31,
circumstance in this case. The evidence on record does not support the 1925. As further certified by CENRO of the DENR, the entire area of
conclusions of both the trial court and the Court of Appeals. Petitioner Lot 806 is an agricultural land; within an alienable and disposable
claimed in its opposition to the application of Manna Properties that, zone; not within a reservation area nor within a forest zone; and does
as a private corporation, Manna Properties is disqualified from holding not encroach upon an established watershed, riverbed, and riverbank
alienable lands of the public domain, except by lease. Petitioner cites protection. Petitioners were thus able to successfully meet the requisite
the constitutional prohibition in Section 3 of Article XII in the 1987 for original registration of title, to wit: open, continuous, exclusive and
Constitution. Petitioner also claims that the land in question is still part notorious possession and occupation of an alienable and disposable
of the public domain. On the other hand, Manna Properties claims that land under a bona fide claim of ownership since June 12, 1945 or
it has established that the land in question has been in the open and earlier.
exclusive possession of its predecessors-in-interest since the 1940s.
Thus, the land was already private land when Manna Properties
acquired it from its predecessors-in-interest.
ADDITIONAL CASES IN NATURAL RESOURCES & ENVIRONMENTAL LAW WITH LAND TITLES AND DEEDS [CASE DIGESTS] JAMES BRYAN DEANG
While it was held that while the best evidence to identify a piece of Intestate Estate of the Late Don Mariano San Pedro vs.
land for registration purposes is the original tracing cloth plan from the Court of Appels
Bureau of Lands, blue print copies and other evidence could also G.R. No. 103727, December 18, 1996
provide sufficient identification. Lot 806 was sufficiently identified by
the blue print copy of the plan and the technical description thereof FACTS: The most fantastic land claim in the history of
both approved by Land Management Services, DENR. the Philippines is the subject of controversy in these two consolidated
cases. The heirs of the late Mariano San Pedro y Esteban laid claim
Republic vs. Hubilla and have been laying claim to the ownership of, against third persons
G.R. No. 157683, February 11, 2005 and the Government itself, a total land area of approximately 173,000
hectares or 214,047 quiniones, on the basis of a Spanish title, entitled
FACTS: Respondents filed an application for registration of title for Titulo de Propriedad Numero 4136 dated April 25, 1894. The claim,
Lot No. 6218-B. Respondents alleged that they have been in open, according to the San Pedro heirs, appears to cover lands in the
continuous, public, peaceful and notorious possession and occupation provinces of Nueva Ecija, Bulacan, Rizal, Laguna and Quezon; and
of the Property, by themselves and their predecessors-in-interest, prior such Metro Manila cities as Quezon City, Caloocan City, Pasay City,
to June 12, 1945. Among others, the respondents presented the City of Pasig and City of Manila, thus affecting in general lands
following documents to support their application: extending from Malolos, Bulacan to the City Hall of Quezon City and
the land area between Dingalan Bay in the north and Tayabas Bay in
1. a blue print copy of the subdivision plan approved by the the south.
Director of Lands through Assistant Regional Director
Ernesto Viquiera; Considering the vastness of the land claim, innumerable disputes
2. a technical description approved by the Land Management cropped up and land swindles and rackets proliferated resulting in
Bureau of the DENR; tedious litigation in various trial courts, in the appellate court and in
3. a certification from the DENR CENRO which states that the the Supreme Court.
Property is entirely within the alienable and disposable zone
as of December 31, 1925 and has not been previously titled; ISSUES:
4. a report of the Land Management Bureau stating that the 1. Whether or not the lower court, acting as a probate court, in
Property is not covered by any previous land registration the petition for letters of administration, committed grave
case; abuse of discretion amounting to lack of jurisdiction in
5. tax declarations dating from 1999 back to 1945 in the names settling the issue of ownership of the San Pedro estate
of Mateo Abrigo and Rodrigo Abrigo after the formers covered by Titulo Propriedad No. 4136.
death. 2. Whether or not petitioners Titulo de Propriedad No. 4136 is
null and void and of no legal force and effect.
The trial court rendered its decision approving the respondents
application for registration. The petitioner appealed the trial courts HELD:
decision to the Court of Appeals. The petitioner, through the Solicitor 1. NO.
General, argued that the trial court erred when it granted the application A probate courts jurisdiction is not limited to the determination of who
despite the failure of respondents to submit the original tracing cloth the heirs are and what shares are due them as regards the estate of a
plan and prove that they complied with the period of possession and deceased person. Neither is it confined to the issue of the validity of
occupation required by law. wills. We held in the case of Maingat v. Castillo, that the main function
of a probate court is to settle and liquidate the estates of deceased
ISSUE: Whether or not the submission of a blueprint copy of the persons either summarily or through the process of administration.
survey plan is a sufficient compliance with the requirements under the Thus, its function necessarily includes the examination of the
Property Registration Decree. properties, rights and credits of the deceased so as to rule on whether
or not the inventory of the estate properly included them for purposes
HELD: YES. of distribution of the net assets of the estate of the deceased to the
While the petitioner correctly asserts that the submission in evidence lawful heirs. Parenthetically, questions of title pertaining to the
of the original tracing cloth plan, duly approved by the Bureau of determination prima facie of whether certain properties ought to be
Lands, is a mandatory requirement, this Court has recognized instances included or excluded from the inventory and accounting of the estate
of substantial compliance with this rule. In previous cases, this Court subject of a petition for letters of administration, as in the intestate
ruled that blueprint copies of the original tracing cloth plan from the proceedings of the estate of the late Mariano San Pedro y Esteban,
Bureau of Lands and other evidence could also provide sufficient maybe resolved by the probate court.
identification to identify a piece of land for registration purposes. The
Court concluded that the subject property was sufficiently identified. 2. YES.
The respondents also filed a motion to admit original tracing cloth plan It is settled that by virtue of Presidential Decree No. 892 which took
with the Court of Appeals during the pendency of the appeal and effect on February 16, 1976, the system of registration under the
attached thereto the original plan. We likewise note that the original Spanish Mortgage Law was abolished and all holders of Spanish titles
tracing cloth plan submitted to the Court of Appeals is the same as the or grants should cause their lands covered thereby to be registered
blueprint subdivision plan offered as evidence before the trial court. under the Land Registration Act within six (6) months from the date of
effectivity of the said Decree or until August 16, 1976. Otherwise, non-
compliance therewith will result in a re-classification of their lands.
Spanish titles can no longer be countenanced as indubitable evidence
of land ownership.

In both cases, petitioners-heirs did not adduce evidence to show that


Titulo de Propriedad 4136 was brought under the operation of P.D. 892
despite their allegation that they did so on August 13, 1976. Time and
again we have held that a mere allegation is not evidence and the party
ADDITIONAL CASES IN NATURAL RESOURCES & ENVIRONMENTAL LAW WITH LAND TITLES AND DEEDS [CASE DIGESTS] JAMES BRYAN DEANG
who alleges a fact has the burden of proving it. Proof of compliance agricultural land. Therefore, with respect to agricultural lands, any
with P.D. 892 should be the Certificate of Title covering the land possession prior to the declaration of the alienable property as
registered. disposable may be counted in reckoning the period of possession to
perfect title under the Public Land Act and the Property Registration
In the petition for letters of administration, it was a glaring error on the Decree.
part of Judge Bagasao who rendered the reconsidered Decision dated
April 25, 1978 to have declared the existence, genuineness and Discussion on CA No. 141 or the Public Land Act
authenticity of Titulo de Propriedad No. 4136 in the name of the CA No. 141 governed the classification and disposition
deceased Mariano San Pedro y Esteban despite the effectivity of P.D. of lands of the public domain. The President is
No. 892. Judge Fernandez, in setting aside Judge Bagasao's decision, authorized, from time to time, to classify the lands of
emphasized that Titulo de Propriedad No. 4136, under P.D. 892, is the public domain into alienable and disposable, timber,
inadmissible and ineffective as evidence of private ownership in the or mineral lands.
special proceedings case.
Alienable and disposable lands of the public domain
Heirs of Mariano Malabanan vs. Republic are further classified according to their uses into
G.R. No. 179987, April 29, 2009 (a) agricultural;
(b) residential, commercial, industrial, or for similar
FACTS: Mario Malabanan filed an application for land registration productive purposes;
covering a parcel of land. Malabanan claimed that he had purchased (c) educational, charitable, or other similar purposes; or
the property from Eduardo Velazco, and that he and his predecessors- (d) reservations for town sites and for public and quasi-
in-interest had been in open, notorious, and continuous adverse and public uses.
peaceful possession of the land for more than thirty (30) years. Apart
from presenting documentary evidence, Malabanan himself and his Q: May a private person validly seek the registration in his/her
witness, Aristedes Velazco, testified at the hearing. Velazco testified name of alienable and disposable lands of the public domain?
that the property originally belonged to a 22 hectare property owned A: Section 11 of the Public Land Act acknowledges that public
by his great-grandfather Lino who had four sons: Benedicto, Gregorio, lands suitable for agricultural purposes may be disposed of by
Eduardo and Esteban, Aristedes grandfather. Upon Linos death, his confirmation of imperfect or incomplete titles through judicial
four sons inherited the property. In 1966, Estebans wife Magdalena legalization.
had become the administrator of all the properties inherited by the
Velazco brothers. After the death of Esteban and Magdalena, their son Section 48(b) of the Public Land Act, as amended by P.D. No.
Virgilio succeeded them in administering the properties, 1073, supplies the details and unmistakably grants that right,
including Lot 9864-A, which originally belonged to his uncle, subject to the requisites stated therein.
Eduardo. It was this property that was sold by Eduardo Velazco to
Malabanan. Section 48(b) of Com. Act No. 141 was amended by
P.D. No. 1073. Two significant amendments were
Assistant Provincial Prosecutor Jose Velazco, Jr. did not cross- introduced by P.D. No. 1073.
examine Aristedes Velazco. The Republic of the Philippines likewise
did not present any evidence to controvert the application.The RTC 1. The term agricultural lands was changed to alienable
rendered judgment in favor of Malabanan. and disposable lands of the public domain.
o The OSG submits that this amendment
The Republic interposed an appeal to the Court of Appeals, arguing restricted the scope of the lands that may be
that Malabanan had failed to prove that the property belonged to the registered. This is not actually the case.
alienable and disposable land of the public domain, and that the RTC o Under Section 9 of the Public Land Act,
had erred in finding that he had been in possession of the property in agricultural lands are a mere subset of lands
the manner and for the length of time required by law for confirmation of the public domain alienable or open to
of imperfect title. The Court of Appeals reversed the RTC dismissing disposition. Evidently, alienable and
the application of Malabanan. The appellate court held that under disposable lands of the public domain are
Section 14(1) of the Property Registration Decree any period of a larger class than only agricultural lands.
possession prior to the classification of the lots as alienable and
disposable was inconsequential and should be excluded from the 2. The length of the requisite possession was changed
computation of the period of possession. Thus, the appellate court from possession for thirty (30) years immediately
noted that since the CENRO-DENR certification had verified that the preceding the filing of the application to possession
property was declared alienable and disposable only on 15 March since June 12, 1945 or earlier.
1982, the Velazcos possession prior to that date could not be factored
in the computation of the period of possession. The CAs decision was Notwithstanding the passage of the Property
based on Herbieto case. Registration Decree and the inclusion of Section 14(1)
therein, the Public Land Act has remained in effect.
Malabanan died while the case was pending. Hence, it was his heirs
who appealed the decision of the appellate court. Petitioners, before Comparison: Sec. 48 of CA No. 141 & Sec. 14 of PD 1549
this Court, rely on our ruling in Republic v. Naguit, which was handed It is clear that Section 48 of the Public Land Act is more
down just four months prior to Herbieto. Petitioners suggest that the descriptive of the nature of the right enjoyed by the possessor than
discussion in Herbieto cited by the Court of Appeals is actually obiter Section 14 of the Property Registration Decree, which seems to
dictum since the Metropolitan Trial Court therein which had directed presume the pre-existence of the right, rather than establishing the
the registration of the property had no jurisdiction in the first place right itself for the first time.
since the requisite notice of hearing was published only after the
hearing had already begun. Naguit, petitioners argue, remains the
controlling doctrine, especially when the property in question is
ADDITIONAL CASES IN NATURAL RESOURCES & ENVIRONMENTAL LAW WITH LAND TITLES AND DEEDS [CASE DIGESTS] JAMES BRYAN DEANG
Section 14(a) of the Property Registration Decree recognizes the becomes patrimonial may be counted for the purpose of completing
substantive right granted under Section 48(b) of the Public Land the prescriptive period. Possession of public dominion property
Act, as well provides the corresponding original registration before it becomes patrimonial cannot be the object of prescription
procedure for the judicial confirmation of an imperfect or according to the Civil Code. As the application for registration
incomplete title. under Section 14(2) falls wholly within the framework of
prescription under the Civil Code, there is no way that possession
There is another limitation to the right granted under Section during the time that the land was still classified as public dominion
48(b). Section 47 of the Public Land Act limits the period within property can be counted to meet the requisites of acquisitive
which one may exercise the right to seek registration under prescription and justify registration.
Section 48. The provision has been amended several times, most
recently by Rep. Act No. 9176 in 2002. ISSUE: Are we being inconsistent in applying divergent rules for
Section 14(1) and Section 14(2)?
Accordingly under the current state of the law, the substantive
right granted under Section 48(b) may be availed of only until 31 HELD: NO.
December 2020. There is no inconsistency. Section 14(1) mandates registration on
the basis of possession, while Section 14(2) entitles registration on
the basis of prescription. Registration under Section 14(1) is
ISSUE: Whether or not for one to acquire the right to seek registration extended under the aegis of the Property Registration Decree and
of an alienable and disposable land of the public domain, it is not the Public Land Act while registration under Section 14(2) is made
enough that the applicant and his/her predecessors-in-interest be in available both by the Property Registration Decree and the Civil
possession under a bona fide claim of ownership since 12 June 1945; Code.
the alienable and disposable character of the property must have been
declared also as of 12 June 1945.
Del Rosario vs. Republic
HELD: NO. G.R. No. 148338, June 6, 2002
The Court declares that the correct interpretation of Section 14(1) is
that which was adopted in Naguit. The contrary pronouncement FACTS: Petitioner filed an application for registration of a parcel of
in Herbieto, as pointed out in Naguit, absurdly limits the application of land. In his application, petitioner stated that he is a Filipino, married
the provision to the point of virtual inutility since it would only cover to Agustina Catalasan, and a resident of Poblacion, Ternate, Cavite;
lands actually declared alienable and disposable prior to 12 June 1945, that he and his predecessors-in-interest had been in the open,
even if the current possessor is able to establish open, continuous, continuous, exclusive, and notorious possession and occupation of the
exclusive and notorious possession under a bona fide claim of land in question, which was alienable and disposable land, under a
ownership long before that date. bona fide claim of ownership since the 1920s or even earlier; and that
such land was being occupied and cultivated by him and his family.
Moreover, the Naguit interpretation allows more possessors under Petitioner further alleged that there was no mortgage or encumbrance
a bona fide claim of ownership to avail of judicial confirmation of their on the land; that the same was not bound by any public or private road
imperfect titles than what would be feasible under Herbieto. This or by any river or creek; and that there was no person having any
balancing fact is significant, especially considering our forthcoming interest therein, legal or equitable, or having possession thereof other
discussion on the scope and reach of Section 14(2) of the Property than himself. Petitioner indicated the owners/claimants/occupants of
Registration Decree. the adjoining properties and annexed to his application the pertinent
documents. On the day he filed his application, petitioner also
Discussion on Section 14 (2) submitted to the Branch Clerk of Court the original tracing cloth plan.
Section 48(b) of the Public Land Act, as amended by Rep. Act No. The clerk of court transmitted to the Land Registration Authority
1942, did not refer to or call into application the Civil Code (LRA) the duplicate copy of petitioner's application for registration of
provisions on prescription. It merely set forth a requisite thirty-year title, the original tracing cloth plan, and the other documents submitted
possession period immediately preceding the application for by petitioner in support of his application.
confirmation of title, without any qualification as to whether the
property should be declared alienable at the beginning of, and During the initial hearing, no oppositor appeared except for the
continue as such, throughout the entire thirty-(30) years. There is provincial prosecutor of Maragondon, Cavite, who appeared on behalf
neither statutory nor jurisprudential basis to assert Rep. Act No. of the Solicitor General. Thereafter, the case was referred to a trial
1942 had mandated such a requirement,[38] similar to our earlier commissioner for the reception of further evidence. Aside from
finding with respect to the present language of Section 48(b), which himself, petitioner presented Raymundo Telia before the trial
now sets 12 June 1945 as the point of reference. commissioner to prove his claim of ownership and title over the parcel
of land applied for registration. Both of them were subjected to cross-
Then, with the repeal of Rep. Act No. 1942, the thirty-year examination by the provincial prosecutor. Petitioner claimed he and
possession period as basis for original registration became Section his family planted in the subject lot kakawati trees, mango and bamboo
14(2) of the Property Registration Decree, which entitled those who trees and raised animals on it. Petitioner testified that he inherited the
have acquired ownership over private lands by prescription under land from his grandfather and he said that he possessed the subject
the provisions of existing laws to apply for original registration. property from 1984, the time the cadastral survey was made thereon,
Again, the thirty-year period is derived from the rule on but also claimed that the first survey on the land was made in 1930.
extraordinary prescription under Article 1137 of the Civil Code. At The trial court rendered its decision granting the application.
the same time, Section 14(2) puts into operation the entire regime Respondent appealed to the Court of Appeals, putting in issue the
of prescription under the Civil Code, a fact which does not hold true failure of petitioner to submit in evidence the original tracing cloth plan
with respect to Section 14(1). and to establish that he and his predecessors-in-interest had been in
open, continuous, and notorious possession of the land applied for
The limitation imposed by Article 1113 dissuades us from ruling registration for the period required by law. The Court of Appeals
that the period of possession before the public domain land rendered its decision reversing the decision of the trial court.
ADDITIONAL CASES IN NATURAL RESOURCES & ENVIRONMENTAL LAW WITH LAND TITLES AND DEEDS [CASE DIGESTS] JAMES BRYAN DEANG
During the hearing, Carandang appeared as oppositor. The trial court
ISSUE: Whether or not THE DENIAL OF PETITIONER'S gave Carandang until 29 November 1999 within which to file his
APPLICATION FOR ORIGINAL REGISTRATION WAS written opposition. Carandang failed. The trial court reinstated the
UNJUSTIFIED and IN THE INTEREST OF JUSTICE, THE Order of General Default. During the hearings, TAN presented three
PROCEEDINGS SHOULD HAVE BEEN REOPENED TO ADMIT witnesses (Torres, Evangelista, and Marquez). The testimonies of
THE ORIGINAL TRACING CLOTH PLAN IN EVIDENCE, TO respondents witnesses showed that Dimayuga had peaceful, adverse,
AVOID A REPETITION OF THE SAME PROCEEDINGS open, and continuous possession of the land in the concept of an owner
ALREADY HAD IN THIS APPLICATION. since 1942. Upon his death, Dimayuga was succeeded by his son
Antonio who executed a Deed of Donation covering the land in favor
HELD: NO. First. Petitioner claims that he should not be faulted for of one of his children, Fortunato. Later, however, Antonio gave
such failure since he turned over the same to the trial court on the day Fortunato another piece of land. Hence, Antonio executed a Partial
he filed his application, but it was submitted to the LRA by the branch Revocation of Donation, and the land was adjudicated to one of
clerk of court and could not be produced during the trial. The Antonios children, Prospero, who sold the land to respondent.
submission in evidence of the original tracing cloth plan, duly
approved by the Bureau of Lands, in cases for application of original ISSUES:
registration of land is a mandatory requirement. The reason for this rule 1. Whether the land is alienable and disposable;
is to establish the true identity of the land to ensure that it does not 2. Whether respondent or its predecessors-in-interest had open,
overlap a parcel of land or a portion thereof already covered by a continuous, exclusive, and notorious possession and
previous land registration, and to forestall the possibility that it will be occupation of the land in the concept of an owner since June
overlapped by a subsequent registration of any adjoining land. The 1945 or earlier; and
failure to comply with this requirement is fatal to petitioner's 3. Whether respondent is qualified to apply for registration of
application for registration. Petitioner contends, however, that he had the land under the Public Land Act.
submitted the original tracing cloth plan to the branch clerk of court,
but the latter submitted the same to the LRA. This claim has no merit. HELD:
Petitioner is duty bound to retrieve the tracing cloth plan from the LRA 1. NO.
and to present it in evidence in the trial court. In this case, respondent submitted two certifications issued by the
DENR. The first Certification by the CENRO certified that lot 10705
Although in one case, it was ruled that a mere blueprint copy of the falls within the ALIENABLE AND DISPOSABLE ZONE under
cloth plan, together with the lot's technical description, was sufficient Project No. 30, Land Classification Map No. 582. The second
to identify the land applied for registration, both the blueprint copy and certification issued by the Forest Management Services of the DENR
the technical description were certified as to their correctness by the (FMS-DENR), stated the same. The certifications are not sufficient.
Director of Lands. In this case, what was marked in evidence, the
advance survey plan and the technical description, lacked the DENR Administrative Order (DAO) No. 20-1988 which was amended
necessary certification from the Bureau of Lands. by DAO No. 38 retained the authority of the CENRO to issue
certificates of land classification status for areas below 50 hectares, as
As to the adverse possession: well as the authority of the PENRO to issue certificates of land
Failed to establish. Although petitioner claims that he possessed Lot classification status for lands covering over 50 hectares. In this case,
No. 1891 by himself and through his predecessors-in-interest since the respondent applied for registration of Lot 10705-B which is over 50
1930s, his tax declaration and tax payment receipt belie the same. It is hectares. The CENRO certificate is beyond the authority of the
noteworthy that the land subject of the application was declared for CENRO to certify as alienable and disposable. Also, the Regional
taxation purposes only on September 8, 1997 and the taxes due thereon Technical Director, FMS-DENR, has no authority under DAO Nos. 20
covered only a period of 10 years beginning 1988 and was paid only and 38 to issue certificates of land classification.
on September 9, 1997, or a little more than a month prior to the filing
of the application. There is no other tax declaration or receipt for tax The applicant for land registration must prove that the DENR Secretary
payments by petitioner's predecessors-in-interest. Moreover, tax had approved the land classification and released the land of the public
declarations and receipts are not conclusive evidence of ownership but domain as alienable and disposable, and that the land subject of the
are merely indicia of a claim of ownership. As petitioner's application application for registration falls within the approved area per
was led only on October 13, 1997, almost 26 years from the time one verification through survey by the PENRO or CENRO. In addition, the
portion was certified as alienable and disposable and 14 years from the applicant for land registration must present a copy of the original
time the remaining portion was certified, the property was still classification approved by the DENR Secretary and certified as a true
unclassified at the time petitioner and his predecessors-in-interest copy by the legal custodian of the official records. These facts must be
allegedly began their possession of the same. established to prove that the land is alienable and disposable. We agree
with petitioner that while the certifications submitted by respondent
Republic vs. T.A.N. Properties show that under the Land Classification Map No. 582, the land became
G.R. No. 154953, June 26, 2008 alienable and disposable on 31 December 1925, the blue print plan
states that it became alienable and disposable on 31 December 1985.
FACTS: This case originated from an Application for Original
Registration of Title filed by T.A.N. Properties, Inc. 2. NO.
covering Lot 10705-B located in Batangas. The trial court set the case We find Evangelistas uncorroborated testimony insufficient to prove
for initial hearing and Notice of Initial Hearing was published and that respondents predecessors-in-interest had been in possession of
posted in the required conspicuous places. All adjoining owners and the land in the concept of an owner for more than 30 years. We cannot
all government agencies and offices concerned were notified of the consider the testimony of Torres as sufficient corroboration. While
initial hearing. There was no oppositor other than the Republic. The Torres claimed to be related to the Dimayugas, his knowledge of their
trial court issued an Order of General Default against the whole world possession of the land was hearsay. He did not even tell the trial court
except as against petitioner. where he obtained his information.

ADDITIONAL CASES IN NATURAL RESOURCES & ENVIRONMENTAL LAW WITH LAND TITLES AND DEEDS [CASE DIGESTS] JAMES BRYAN DEANG
The tax declarations presented were only for the years starting 1955. Herein respondents appealed the foregoing decision to the CA which
While tax declarations are not conclusive evidence of ownership, they promulgated its Decision, the dispositive portion of which is
constitute proof of claim of ownership. The payment of the realty taxes reproduced hereunder:
starting 1955 gives rise to the presumption that the Dimayugas claimed
ownership or possession of the land only in that year. THE FOREGOING CONSIDERED, the contested Decision while
AFFIRMED is hereby MODIFIED. The heirs of Emiliano Aying,
3. NO Simeon Aying and Roberta Aying are hereby declared as the lawful
The 1987 Constitution absolutely prohibits private corporations from owners of the contested property but equivalent only to 3/8.
acquiring any kind of alienable land of the public domain. In this case,
respondent acquired the land on 8 August 1997 from Prospero, who, ISSUES:
along with his predecessors-in-interest, has not shown to have been, as 1. whether or not respondents' cause of action is
of that date, in open, continuous, and adverse possession of the land imprescriptible; and
for 30 years since 12 June 1945. In short, when respondent acquired 2. if their right to bring action is indeed imprescriptible, may
the land from Prospero, the land was not yet private property. the principle of laches apply.

The length of possession of the land by the corporation cannot be HELD:


tacked on to complete the statutory 30 years acquisitive prescriptive Respondents alleged in their amended complaint that not all the co-
period. Only an individual can avail of such acquisitive prescription owners of the land in question signed or executed the document
since both the 1973 and 1987 Constitutions prohibit corporations from conveying ownership thereof to petitioner and made the conclusion
acquiring lands of the public domain. that said document is null and void. We agree with the ruling of the
RTC and the CA that the Extra-Judicial Partition of Real Estate with
Aznar Brothers Realty Co. vs. Aying Deed of Absolute Sale is valid and binding only as to the heirs who
G.R. No. 144773, May 16, 2005 participated in the execution thereof, hence, the heirs of Emiliano,
Simeon and Roberta Aying, who undisputedly did not participate
FACTS: The disputed property is Lot No. Crisanta Maloloy-on therein, cannot be bound by said document. However, the facts on
petitioned for the issuance of a cadastral decree in her favor over said record show that petitioner acquired the entire parcel of land with the
parcel of land. After her death in 1930, the Cadastral Court issued a mistaken belief that all the heirs have executed the subject document.
Decision directing the issuance of a decree in the name of Crisanta Thus, the trial court is correct that the provision of law applicable to
Maloloy-on's eight children, namely: Juan, Celedonio, Emiliano, this case is Article 1456 of the Civil Code which states:
Francisco, Simeon, Bernabe, Roberta and Fausta, all surnamed Aying.
The certificate of title was, however, lost during the war. Subsequently, ART. 1456. If property is acquired through mistake or fraud, the
all the heirs of the Aying siblings executed an Extra-Judicial Partition person obtaining it is, by force of law, considered a trustee of an
of Real Estate with Deed of Absolute Sale dated March 3, 1964, implied trust for the benefit of the person from whom the property
conveying the subject parcel of land to Aznar Brothers Realty comes.
Company. Said deed was registered with the Register of Deeds of
Lapu-Lapu City under Act No. 3344 (the law governing registration The rule that a trustee cannot acquire by prescription ownership over
for unregistered land). property entrusted to him until and unless he repudiates the trust,
applies to express trusts and resulting implied trusts. However, in
In 1988, herein petitioner filed a Petition for Reconstitution of the constructive implied trusts, prescription may supervene even if the
Original Title. The court granted said petition. In 1991, petitioner, trustee does not repudiate the relationship. Necessarily, repudiation of
claiming to be the rightful owner of the subject property, sent out said trust is not a condition precedent to the running of the prescriptive
notices to vacate, addressed to persons occupying the property. period.
Unheeded, petitioner then filed a complaint for ejectment. The MTC
ordered the occupants to vacate the property. The case eventually As to prescriptive period
reached this Court which Decision was promulgated in favor of herein In the present case, respondents Wenceslao Sumalinog, an heir of
petitioner, declaring it as the rightful possessor of the parcel of land in Roberta Aying; Laurencio Aying, an heir of Emiliano Aying; and
question. Paulino Aying, an heir of Simeon Aying, all testified that they had
never occupied or been in possession of the land in dispute. Hence, the
Meanwhile, herein respondents, along with other persons claiming to prescriptive period of ten years would apply to herein respondents. In
be descendants of the eight Aying siblings, (220) persons, had filed a this case, since the Extra-Judicial Partition of Real Estate with Deed of
complaint for cancellation of the Extra-Judicial Partition with Absolute Absolute Sale was registered under Act No. 3344 and not under Act
Sale, recovery of ownership, injunction and damages with the RTC. No. 496, said document is deemed not registered. Accordingly, the ten-
The complaint was dismissed twice without prejudice. Said complaint year prescriptive period cannot be reckoned from March 6, 1964, the
was re-filed on August 19, 1993. In their amended complaint, herein date of registration of the subject document under Act No. 3344. The
respondents alleged that: they are co-owners being descendants of the prescriptive period only began to run from the time respondents had
registered owners; they had been in actual, peaceful, physical, open, actual notice of the Extra-Judicial Partition of Real Estate with Deed
adverse, continuous and uninterrupted possession disturbed only in the of Absolute Sale.
last quarter of 1991 when some of them received notices to vacate from
petitioner. They also claimed that the extra-judicial partition of real In the case at bar, it was petitioner, as the defendant before the RTC,
estate with deed of absolute sale is a fraud and is null and void ab initio. which set up in its Answer the affirmative defense of prescription. It
After trial, the RTC rendered a Decision ruling that respondents' was, therefore, incumbent upon petitioner to prove the date from which
evidence failed to prove that the extra-judicial partition with deed of the prescriptive period began to run.
absolute sale was a totally simulated or fictitious contract and
concluded that said document is valid, thus, effectively conveying to Respondents filed their Amended Complaint on December 6, 1993.
petitioner the property in question. Thus, with regard to respondent heirs of Roberta Aying who had
knowledge of the conveyance as far back as 1967, their cause of action
is already barred by prescription when said amended complaint was
ADDITIONAL CASES IN NATURAL RESOURCES & ENVIRONMENTAL LAW WITH LAND TITLES AND DEEDS [CASE DIGESTS] JAMES BRYAN DEANG
filed as they only had until 1977 within which to bring action. As to submerged areas. Only then can these lands qualify as agricultural
the respondent heirs of Emiliano and Simeon Aying, they were able to lands of the public domain, which are the only natural resources the
initiate their action for reconveyance of property based on implied or government can alienate. In their present state, the 592.15 hectares of
constructive trust well within the ten-year prescriptive period reckoned submerged areas are inalienable and outside the commerce of man.
from 1991 when they were sent by petitioner a notice to vacate the
subject property. 3. Since the Amended JVA seeks to transfer to AMARI, a private
corporation, ownership of 77.34 hectares of the Freedom Islands, such
Evidently, laches cannot be applied against respondent heirs of transfer is void for being contrary to Section 3, Article XII of the 1987
Emiliano and Simeon Aying, as they took action to protect their Constitution which prohibits private corporations from acquiring any
interest well within the period accorded them by law. kind of alienable land of the public domain.

Chavez vs. PEA 4. Since the Amended JVA also seeks to transfer to AMARI ownership
G.R. No. 133250, July 9, 2002 of 290.156 hectares of still submerged areas of Manila Bay, such
transfer is void for being contrary to Section 2, Article XII of the 1987
FACTS: In 1973, the Commissioner on Public Highways entered into Constitution which prohibits the alienation of natural resources other
a contract to reclaim areas of Manila Bay with the Construction and than agricultural lands of the public domain.
Development Corporation of the Philippines (CDCP). PEA (Public
Estates Authority) was created by President Marcos under P.D. 1084, PEA may reclaim these submerged areas. Thereafter, the government
tasked with developing and leasing reclaimed lands. These lands were can classify the reclaimed lands as alienable or disposable, and further
transferred to the care of PEA under P.D. 1085 as part of the Manila declare them no longer needed for public service. Still, the transfer of
Cavite Road and Reclamation Project (MCRRP). CDCP and PEA such reclaimed alienable lands of the public domain to AMARI will be
entered into an agreement that all future projects under the MCRRP void in view of Section 3, Article XII of the 1987Constitution which
would be funded and owned by PEA. prohibits private corporations from acquiring any kind of alienable
land of the public domain.
By 1988, President Aquino issued Special Patent No. 3517 transferring
lands to PEA. It was followed by the transfer of three Titles (7309, Alabang Development Corp. vs. Valenzuela
7311 and 7312) by the Register of Deeds of Paranaque to PEA G.R. No. L-54094, August 30, 1982
covering the three reclaimed islands known as the FREEDOM
ISLANDS. Subsquently, PEA entered into a joint venture agreement FACTS: Petitioners filed this petition against the decision of the CFI
(JVA) with AMARI, a Thai-Philippine corporation to develop the of Pasay City, Branch XXIX granting the reconstitution of the
Freedom Islands. Along with another 250 hectares, PEA and AMARI certificate of title, original land owner's duplicate copy in the name of
entered the JVA which would later transfer said lands to AMARI. This Manuela Aquial supposedly covering Lots 2 and 4 of Survey Plan 11-
caused a stir especially when Sen. Maceda assailed the agreement, 4374 on the ground that the said decision suffers from jurisdictional
claiming that such lands were part of public domain (famously known infirmity fatal to the petition for reconstitution since said petition-and
as the mother of all scams). the notice thereof lacked essential data mandatorily required by law
(Secs. 12-13 R. A. 26) and, that the said decision was invalidly issued
Peitioner Frank J. Chavez filed case as a taxpayer praying for without actual and personal notice having been served upon
mandamus, a writ of preliminary injunction and a TRO against the sale possessors, actual occupants and adjoining owners of the property
of reclaimed lands by PEA to AMARI and from implementing the involved who are indispensable parties in interest and without whom a
JVA. Following these events, under President Estradas admin, PEA valid judgment cannot be rendered. Aside from alleging ownership
and AMARI entered into an Amended JVA and Mr. Chaves claim that over the lands in question subsequently sold to innocent purchasers in
the contract is null and void. good faith and for valuable consideration as part of the Alabang Hills
Village, a subdivision which it owned, petitioners claimed that on the
ISSUE: basis of the technical descriptions contained in their titles and as appear
1. W/N the transfer to AMARI lands reclaimed or to be in the alleged title sought to be reconstituted, the latter overlap the
reclaimed as part of the stipulations in the (Amended) JVA parcels owned and registered in their names. A temporary restraining
between AMARI and PEA violate Sec. 3 Art. XII of the 1987 order was issued to restrain respondents from enforcing the challenged
Constitution decision and specifically the Register of Deeds of Rizal from issuing a
2. W/N: the court is the proper forum for raising the issue of reconstituted title in their name.
whether the amended joint venture agreement is grossly
disadvantageous to the government. Respondents, among others, aver that the petition is barred by laches
due to the failure of petitioners to appeal from the trial court's orders
HELD: denying their belated intervention and motion for new trial; that the
On the issue of Amended JVA as violating the constitution: question for ownership over the alleged overlappings cannot be
1. The 157.84 hectares of reclaimed lands comprising the Freedom sweepingly adjudicated in a certiorari proceeding or a reconstitution
Islands, now covered by certificates of title in the name of PEA, are case; and that the existence of their title is established by government
alienable lands of the public domain. PEA may lease these lands to documents.
private corporations but may not sell or transfer ownership of these
lands to private corporations. PEA may only sell these lands to ISSUE: Whether or not the lower court has jurisdiction to act upon the
Philippine citizens, subject to the ownership limitations in the 1987 petition for reconstitution.
Constitution and existing laws.
HELD: Upon examination of the subject petition for reconstitution,
2. The 592.15 hectares of submerged areas of Manila Bay remain the Court noted that some essential data required in Sections 12 and
inalienable natural resources of the public domain until classified as 13, R. A. 26 have been omitted. Neither do these data appear in the
alienable or disposable lands open to disposition and declared no Notice of Hearing such that no adjoining owner, occupant or possessor
longer needed for public service. The government can make such was ever served a copy thereof by registered mail or otherwise.
classification and declaration only after PEA has reclaimed these Following the ruling in the Bernal case, (L-45168, 93 SCRA 238) the
ADDITIONAL CASES IN NATURAL RESOURCES & ENVIRONMENTAL LAW WITH LAND TITLES AND DEEDS [CASE DIGESTS] JAMES BRYAN DEANG
Supreme Court ruled that "said defects have not invested the Court the Proceedings of Reconstitution Case No. 504-P as well as to
with authority or jurisdiction to proceed with the ease because the whether the Court of Appeals erred in sustaining the decision of
manner or mode of obtaining jurisdiction as prescribed by the statute respondent Judge, will find the correct and appropriate answers.
which is mandatory has not been strictly followed, thereby rendering
all proceedings utterly null and void. ISSUE: Whether or not the trial court properly acquired and was
invested with jurisdiction to hear and decide Reconstitution Case No.
The Court likewise observed that the lots involved in the reconstitution 504-P in the light of the strict and mandatory provisions of Republic
case are covered by the same survey plan in the Bernal case and they Act No. 26.
are contiguous. Thus the finding as to the non-veracity and falsity of
the survey plan submitted in support of reconstitution in the Bernal HELD:
case is likewise warranted to reject the evidence presented by Upon a cursory reading of both the petition for reconstitution and the
respondents in support of their petition for reconstitution. notice of hearing, it is at once apparent that Tahanan has not been
named, cited or indicated therein as the owner, occupant or possessor
The argument that petitioners are guilty of laches is unavailing for here, of property adjacent to Lot 2, title to which is sought to be
there is no finality of any decision to speak of since the decision under reconstituted. Neither do the petition and the notice state or mention
question is pronounced null and void far having been rendered without that Tahanan is the occupant or possessor of a portion of said Lot 2.
jurisdiction and the judgment of reconstitution is "ineffective'' against The result of this omission or failure is that Tahanan was never notified
the owners of the land covered thereby who were not joined as parties of the petition for reconstitution and the hearings or proceedings
in the proceedings. therein.

The judgment ordering the Register of Deeds, Metro Manila, Makati, That the Pascuals deliberately omitted, concealed or sought to conceal
Branch IV to reconstitute from Decree No. 15170 and the plan and the fact that Tahanan is the owner, occupant and possessor of property
technical descriptions submitted, the alleged certificate of title, original adjacent to the former's alleged property may be deduced by their
and owner's duplicate copy in the name of Manuela Aquial was failure to comply with the order of Judge Leo Medialdea issued in the
annulled and set aside; the petition for reconstitution was ordered original petition for reconstitution wherein "the petitioners are hereby
dismissed; the temporary restraining order made permanent and a copy required to amend their petition, within ten days from receipt hereof,
of the decision at bar was ordered to be furnished the Minister of by indicating therein the names and addresses of all boundary owners
Justice for the institution of appropriate criminal proceedings against of the parcels of land in question as well as the names and addresses of
private respondents and all others who have assisted or conspired with all persons occupying the same." The amended Petition
them as may be warranted by the evidence of record. notwithstanding, the omission of Tahanan as adjoining owner and even
as occupant of portions of the supposed Pascual property is palpable
Tahanan Development Corporation vs. CA and conspicuous. It is all too evident that the Pascuals in refiling their
G.R. No. 55771, November 15, 1982 Petition for Reconstitution had no intention to notify nor give cause for
notification and knowledge to all adjacent or boundary owners,
FACTS: There are three cases recently decided by the Supreme Court particularly Tahanan.
that are directly related to and squarely identified with the petition at
bar. In the first case, Our Resolution admitted the intervention of the The Pascuals are duty-bound to know who are their actual adjacent
intervenors filed before the Supreme Court at the stage of the boundary owners on all sides and directions of their property. They are
proceedings where trial of the petition for judicial reconstitution had charged with the obligation to inquire who their neighbors are in actual
already been concluded, the judgment thereon granting the possession and occupancy not only of portions of their own property
reconstitution had been promulgated and on appeal by the losing party, but also of land adjacent thereto. This duty or obligation cannot be
the same was affirmed by the Court of Appeals and the petition for ignored or simply brushed aside where the location or the properties
certiorari to review said judgment was already submitted for decision involved is a prime site for land development, expansion, suitable for
in the Supreme Court. residential, commercial and industrial purposes and where every
square inch of real estate becomes a valuable and profitable
The second case is Our decision on the merits of the certiorari petition investment. It is of public knowledge in the community of Paraaque
wherein We ruled, among others, that the judgment of the lower court that "Tahanan Village" is a privately owned and occupied residential
granting the petition for judicial reconstitution of Transfer Certificate subdivision, plainly visible to the general public by reason of the
of Title No. 42449 of the Registry of Deeds of Rizal in the name of perimeter fence or wall separating it from adjacent estates the roads
Demetria Sta. Maria Vda. de Bernal covering two parcels of land was and streets therein and leading thereto, the numerous home
null and void for failure to comply with the mandatory requirements of constructions and buildings going on, the visible electrical, lighting
Republic Act No. 26. We further held that TCT No. 42449 was fake and water supply installations, the presence of private security guards
and spurious. thereat and the numerous signs and billboards advertising the estate as
a housing development owned and/or managed by petitioner Tahanan.
In the third case, the Supreme Court directly ruled that the judgment It is preposterous to claim that the area is public land.
of the Court of First Instance of Rizal, Pasay City, Branch XXIX, in
Reconstitution Case No. 504-P Land Registration Case No. 9368 in the We also find that the Notice of Hearing directed that copies thereof be
name of Manuela Aquial, was null and void. posted only in the bulletin board of the Court of First Instance of Pasay
City and no more, whereas the law specifically require that the notice
In the instant appeal before the Court, petitioner Tahanan assigns of the petition shall be posted on the main entrance of the municipality
numerous errors committed by the appellate court but the principal and or city on which the land is situated, at the provincial building and at
fundamental issues to be resolved is whether or not the trial court the municipal building at least 30 days prior to the date of hearing.
properly acquired and was invested with jurisdiction to hear and decide
Reconstitution Case No. 504-P in the light of the strict and mandatory The failure or omission to notify Tahanan as the owner, possessor or
provisions of Republic Act No. 26. Upon resolving this pivotal issue, occupant of property adjacent to Lot 2 or as claimant or person having
the corollary issue as to respondent Judge's grave abuse of discretion an interest, title or claim to a substantial portion (about 9 hectares more
in denying Tahanan's Petition To Set Aside Decision and To Re-Open or less) of Lot 2, as well as the failure or omission to post copies of the
ADDITIONAL CASES IN NATURAL RESOURCES & ENVIRONMENTAL LAW WITH LAND TITLES AND DEEDS [CASE DIGESTS] JAMES BRYAN DEANG
Notice of Hearing on the main entrance of the municipality on which implied amendment of Section 13 of R.A. No. 26 by virtue of Section
the land is situated, at the provincial building and at the municipal 110 of the Decree.
building thereat, are fatal to the acquisition and exercise of jurisdiction
by the trial court. New Durawood Co. Inc. vs. CA
G.R. No. 111732, February 20, 1996
We allowed the intervention of adjacent owners even during the
pendency of the appeal from the decision granting reconstitution, the FACTS: On February 14, 1990, a "Petition for Judicial Reconstitution
appeal then in the Supreme Court, in the paramount interest of justice of the Lost Owner's Duplicate Certicates of TCT Nos. 140486; 156454
and as an exception to Section 2, Rule 12 of the Rules of Court. and 140485" was filed by petitioner-corporation. Attached to said
Petitioner Tahanan having sought to intervene in the court below and petition was an "Affidavit of Loss" of respondent Orlando S. Bongat,
alleging material and substantial interest in the property to which title one of the stockholders of petitioner-corporation. Finding the petition
is sought to be reconstituted, in its Motion To Set Aside Decision and "to be sufficient in form and in substance," respondent Judge set the
Re-Open Proceedings duly verified and attaching therewith xerox case for hearing. Rrespondent Judge issued the questioned order.
copies of its transfer certificates of title of its properties adjoining and Sometime in May, 1991, petitioner discovered that the original TCT
even overlapped by that of the Pascuals to the extent of some 9 hectares Nos. N-140485, N140486 and 156454 on file with the Register of
in area, the trial court ought to have admitted said motion. There was Deeds of Rizal had been cancelled and, in lieu thereof, TCT Nos.
reversible error in refusing to do so. 200100, 200101 and 200102 had been issued in the name of respondent
Durawood construction and Lumber Supply, Inc. Surprised by this
Republic vs. Marasigan cancellation, petitioner after investigation found out about the
G.R. No. 85515, June 6, 1991 reconstitution proceeding in the respondent trial court. So, on July 17,
1991, petitioner filed suit in the Court of Appeals praying for the
FACTS: Petitioner is one of the vendees of a certain parcel of land annulment of the assailed order penned by respondent Judge. It also
situated in Malamig, Calapan, Oriental Mindoro embraced in and prayed for the cancellation of the new certificates (TCT Nos. 200100,
covered by TCT No. T-66062 and registered in the name of Epifania 200101 and 200102). The respondent Court of Appeals rendered the
Alcano as evidenced by a document of sale executed by the registered assailed Decision and on August 30, 1993, the Resolution denying the
owner. The original copy of said title which was usually kept in the motion for reconsideration. Hence, the present recourse to the Supreme
Office of the Register of Deeds of this province was destroyed by Court.
reason of the fire which razed to the ground the entire Capitol Building
then housing said office. It appears further that there are no co-owner's, Petitioner argues that a reconstitution proceeding is one in rem and thus
mortgagee's, lessee's duplicate copy of said certificate of title which jurisdiction can be acquired only through publication and notice sent
had been previously issued by the Register of Deeds of this province; pursuant to Section 13, Republic Act No. 26. It also alleges that fraud
that the petitioner is in actual possession of the area of 16,647 square is manifest (1) from the insufficient allegations of the petition filed
meters which was sold to her and that she is benefitting from the before the trial court, as it (the petition) does not mention the names of
produce of the improvements existing on the area belonging to her." adjoining land owners and interested persons, as well as (2) from the
and disquisition: "Accordingly, finding the instant petition to be well- affidavit of loss attached to the petition.
founded and there being no opposition to its approval, same is hereby
granted. The Register of Deeds of this province is hereby directed to In their Comment, private respondents aver that in 1990, these three
reconstitute the original and the owner's duplicate copies of Transfer lots were sold by petitioner to Durawood Construction and Lumber
Certificate of Title No. T-66062 in the name of the registered owners. Supply, Inc. but the sale in their favor could not be registered because
"the certicates of title . . . were lost." They also allege that the
Petitioner herein, through the Office of the Solicitor General, appealed applicable law is Section 109 of R.A. No. 496, as amended by PD
from said Order to the Court of Appeals arguing that THE TRIAL 1529, and not Sec. 13 of R.A. No. 26, and that fraud, in order to serve
COURT ERRED IN ACQUIRING JURISDICTION OVER THE as basis for the annulment of a judgment "must be extrinsic or
INSTANT PETITION FOR RECONSTITUTION OF THE collateral in character," which is not the case in the action before the
ORIGINAL AND THE OWNER'S DUPLICATE COPIES OF TCT court a quo.
NO. T-66062 WITHOUT THE REQUISITE SERVICE OF NOTICE
OF HEARING TO THE ADJOINING OWNERS AND ACTUAL ISSUES:
OCCUPANTS OF THE LAND AS REQUIRED BY SECTION 13 OF (1) Which law governs the issuance of new owner's duplicate
REPUBLIC ACT NO. 26. Respondent Court of Appeals brushed aside certificates of title in lieu of lost ones?
the arguments of petitioner (2) Did the respondent trial court have jurisdiction to order the
issuance of the new owner's duplicate certificates?
ISSUE: Whether notices to adjoining owners and the actual occupants (3) Was the reconstitution of the said owner's duplicate
of the land are mandatory and jurisdictional in judicial reconstitution certificates of title obtained through fraud?
of certificates of title.
HELD:
HELD: Section 13 of R.A. No. 26 has not been altered, modified or 1. A reading of both provisions clearly shows that Section 109
amended. Since the requirement therein of service of notice of the of PD 1529 is the law applicable in petitions for issuance of
initial hearing to the adjoining owners and the actual occupants of the new owners duplicate certificates of title which are lost or
land was not complied with in this case, the court below did not, stolen or destroyed. On the other hand, R.A. 26 applies only
therefore, acquire jurisdiction over the petition for the reconstitution of in cases of reconstitution of lost or destroyed original
Transfer Certificate of Title No. 66062. Accordingly, the respondent certificates on file with the Register of Deeds.
Court of Appeals gravely erred in arming the Order of the trial court
granting the petition and in holding that said Section 13 has been "at 2. In the instant case, the owner's duplicate certificates of title
least impliedly amended" by Section 23 in relation to Section 110 of were in the possession of Dy Quim Pong, the petitioner's
P.D. No. 1529 which took effect on 11 June 1978. Worse, it committed chairman of the board and whose family controls the
a serious blunder when it used this clause to support its proposition of petitioner-corporation. Since said certificates were not in
fact "lost or destroyed," there was no necessity for the
ADDITIONAL CASES IN NATURAL RESOURCES & ENVIRONMENTAL LAW WITH LAND TITLES AND DEEDS [CASE DIGESTS] JAMES BRYAN DEANG
petition led in the trial court for the "Issuance of New
Owner's Duplicate Certificates of Title . . . ." In fact, the said
court never acquired jurisdiction to order the issuance of new
certificates. Hence, the newly issued duplicates are
themselves null and void. It is obvious that this lapse
happened because private respondents and respondent judge
failed to follow the procedure set forth in PD. No. 1529
which, as already stated, governs the issuance of new
owner's duplicate certificates of title.

3. We are appalled by this rather novel interpretation of


corporate law. It is clear that, there having been no quorum
present during the meeting in question, the board of directors
could not have validly given Gaw any express authority to
file the petition. Upon the other hand, the doctrine of
"apparent authority" cannot apply as to Gaw because, being
a mere branch manager, he could not be looked upon as a
corporate officer clothed with the implied or "apparent"
power to file suit for and in behalf of a corporation. Neither
will estoppel prevent the corporation from questioning
Gaw's acts. Precisely, these acts were hidden from the
company and its top officers. How then can estoppel attach?
Suffice it to say then, that by his surreptitious filing of the
petition for reconstitution without authority express or
implied of his employer, Gaw enabled respondent
corporation to acquire the certificates of title in a manner
contrary to law.

ADDITIONAL CASES IN NATURAL RESOURCES & ENVIRONMENTAL LAW WITH LAND TITLES AND DEEDS [CASE DIGESTS] JAMES BRYAN DEANG

You might also like